[Toán 8] BĐT

0

01263812493

mình chưa biết gõ nên thông cảm
C/m [TEX](1+\frac{1}{a^3})(1+\frac{1}{b^3})(1+\frac{1}{c^3})\geq \frac{729}{512}[/TEX]
trong đó a,b,c là các số thực thoả mãn a+b+c=6

[TEX]\blue VT=1+ \frac{1}{a^3}+ \frac{1}{b^3}+ \frac{1}{c^3}+ \frac{1}{a^3b^3}+\frac{1}{b^3c^3}+ \frac{1}{c^3a^3}+ \frac{1}{a^3b^3c^3} \geq 1+ \frac{3}{abc} + \frac{3}{a^2b^2c^2}+ \frac{1}{a^3b^3c^3}=(1+ \frac{1}{abc})^3 \geq (1+ \frac{1}{(\frac{a+b+c}{3})^3})^3=VP[/TEX]
[TEX]\blue \rightarrow a=b=c=2[/TEX]
Cái này là trường hợp a,b,c >0 nhe em
 
T

thienlong_cuong

mình chưa biết gõ nên thông cảm
C/m [TEX](1+\frac{1}{a^3})(1+\frac{1}{b^3})(1+\frac{1}{c^3})\geq \frac{729}{512}[/TEX]
trong đó a,b,c là các số thực thoả mãn a+b+c=6

Bài này hình như là cái bài mà lực luợng boy bó tay trong trận chiến boy and girl năm ngoái !

Ta có

[TEX](1+\frac{1}{a^3})(1+\frac{1}{b^3})(1+\frac{1}{c^3})[/TEX]

[TEX]= ({1 + \frac{1}{b^3} + \frac{1}{a^3} + \frac{1}{a^3b^3})(1 + \frac{1}{c^3})[/TEX]

[TEX]= 1 + \frac{1}{a^3} + \frac{1}{b^3} + \frac{1}{c^3} + \frac{1}{a^3b^3} + \frac{1}{b^3c^3} + \frac{1}{a^3c^3} + \frac{1}{a^3b^3c^3}[/TEX]

Áp dụng BĐT AM - GM
Ta có !


[TEX]\frac{1}{a^3} + \frac{1}{b^3} + \frac{1}{c^3} \geq 3.\sqrt[3]{\frac{1}{a^3.b^3.c^3}}[/TEX]

[TEX]\frac{1}{a^3b^3} + \frac{1}{b^3c^3} + \frac{1}{a^3c^3} \geq 3.\sqrt[3]{\frac{1}{(abc)^6}}[/TEX]

\Rightarrow [TEX]1 + \frac{1}{a^3} + \frac{1}{b^3} + \frac{1}{c^3} + \frac{1}{a^3b^3} + \frac{1}{b^3c^3} + \frac{1}{a^3c^3} + \frac{1}{a^3b^3c^3} \geq 1 + \frac{3}{abc} + \frac{1}{(abc)^2} + \frac{1}{(a +b+c)^3}[/TEX]

\Rightarrow [TEX]1 + \frac{1}{a^3} + \frac{1}{b^3} + \frac{1}{c^3} + \frac{1}{a^3b^3} + \frac{1}{b^3c^3} + \frac{1}{a^3c^3} + \frac{1}{a^3b^3c^3} \geq (1 + \frac{1}{abc})^3[/TEX] _____(1)

Nhớ mang máng cái BDT

[TEX](a +b+c)^3 \geq a^3 + b^3 + c^3 + 24abc[/TEX]

Áp dụng BDT AM - GM
Mà [TEX]a^3 + b^3 + c^3 \geq 3abc[/TEX]

\Rightarrow [TEX](a +b +c)^3 \geq 27abc[/TEX]

\Rightarrow [TEX] 6^3 \geq 27abc[/TEX]

\Rightarrow [TEX]abc \leq 8[/TEX]

\Rightarrow [TEX]\frac{1}{abc} \geq \frac{1}{8}[/TEX] ______(2)

Áp dụng (1) và (2)

[TEX]1 + \frac{1}{a^3} + \frac{1}{b^3} + \frac{1}{c^3} + \frac{1}{a^3b^3} + \frac{1}{b^3c^3} + \frac{1}{a^3c^3} + \frac{1}{a^3b^3c^3} \geq ( 1 + \frac{1}{abc})^3 \geq ( 1 + \frac{1}{8})^3 = \frac{729}{512}[/TEX] (đpcm)
 
Last edited by a moderator:
Top Bottom